Derivación del operador de posición en QM

Para la definición del operador de cantidad de movimiento

PAG ^ = i
en mecánica cuántica, según tengo entendido, puede derivar esto considerando una definición más general de momento, es decir, "momento canónico", que es un operador y luego aplicar este operador a las funciones de onda. Esto se muestra aquí en esta entrada de wiki . Podemos alternativamente comenzar con las traducciones y usar la cara de que el operador de momento es un generador de traducciones como se hace aquí en esta entrada de wiki .

Lo que me interesa es cuál es la derivación más fundamental para el operador de posición:

X ^ = X .
Hasta aquí he considerado que la motivación para definir el operador de posición proviene de la definición del valor esperado
X = d X X | ψ ( X ) | 2 = ψ | X | ψ
dónde | ψ está normalizado. ¿Es este el alcance total de la motivación o el punto principal a considerar? ¿Existe otra motivación para el operador de posición o se toma de esta motivación y se confirma mediante experimentos que es aceptable?

Creo que esto es más una definición. Los valores de los observables están dados por los valores propios de su operador correspondiente. Por lo tanto, esto es lo que esperaría que hiciera el operador de posición: darle la posición. Equivalentemente, pag por supuesto opera en el pag estados para dar el impulso, S z da la proyección de espín, etc.
Estás confundiendo entre operadores y sus representaciones en (algunas) bases . Las respuestas a continuación abordan correctamente los problemas.
@GennaroTedesco: debido a que se pueden agregar respuestas adicionales de calidad variable más adelante, es posible que no desee publicar un comentario que recomiende "las respuestas a continuación" sin especificar qué respuestas.
Véase, por ejemplo, el teorema de Stone-von Neumann en Wikipedia . Si le gusta esta pregunta, también puede disfrutar leyendo esta publicación de Phys.SE.
@WillO Oh, sí, claro, eso fue una tontería: D.
Relacionado: physics.stackexchange.com/q/45248/2451 y enlaces allí.

Respuestas (3)

X ^ = X porque elegimos trabajar en una base de vectores propios de X ^ , es decir, funciones de onda.

Tenemos un espacio Hilbert H que es un espacio vectorial en el que podemos elegir cualquier base que deseemos. La mayoría de las veces elegimos trabajar en una base que diagonaliza el operador de posición X ^ . Los estados básicos satisfacen

X ^ | X = X | X
Una vez que se elige tal base, cualquier estado | Ψ en H se puede expandir en él, es decir, podemos escribir
| Ψ = d X | X Ψ ( X )
La función Ψ ( X ) se llama función de onda. Podemos invertir esto para encontrar
Ψ ( X ) = X | Ψ

Ahora, ¿cuál es el significado de ( X ^ Ψ ) ( X ) ? Por definición es la función de onda del estado X ^ | Ψ , es decir, es X | X ^ | Ψ . Entonces es inmediatamente obvio que

X | X ^ | Ψ = X X | Ψ = X Ψ ( X )
Así, encontramos que ( X ^ Ψ ) ( X ) = X Ψ ( X ) . Por esta razón, escribimos por conveniencia X ^ = X , pero hay que recordar que esto solo es cierto en la base de coordenadas.

Operador de cantidad de movimiento en base coordinada: entonces podría preguntar cómo se puede derivar la expresión para el operador de cantidad de movimiento PAG ^ en la base de coordenadas. Esto se hace de la siguiente manera.

Por definición ( PAG ^ Ψ ) ( X ) = X | PAG ^ | Ψ . Entonces,

( PAG ^ Ψ ) ( X ) = d pag 2 π X | pag pag | PAG ^ | Ψ = d pag 2 π X | pag pag | Ψ pag = d X d pag 2 π X | pag pag | X X | Ψ pag
A continuación, usando X | pag = mi i pag X , tenemos
( PAG ^ Ψ ) ( X ) = d X d pag 2 π mi i pag ( X X ) pag Ψ ( X )
ahora escribimos
mi i pag ( X X ) pag = i X mi i pag ( X X )
Usando esto, podemos realizar explícitamente la integral sobre pag y encontramos
( PAG ^ Ψ ) ( X ) = i X Ψ ( X )
Por lo tanto, en base a coordenadas, escribimos
PAG ^ = i X

PD: por supuesto, somos libres de elegir cualquier base que queramos. Podríamos, por ejemplo, trabajar en base al impulso. En esta base, tendríamos

PAG ^ = pag   , X ^ = i pag   .

¿Hay alguna posibilidad de que podamos discutir esto en el chat? El punto de partida de su definición es
X ^ | X = X | X .
El pensamiento aquí es que comenzamos con el postulado de QM que establece que "cualquier observable es un operador hermitiano que tiene estados propios que son la base del espacio de Hilbert". No tiene sentido no empezar desde ese punto, ya que en las dos primeras ecuaciones tuyas ya asumiste que | X es un estado propio de un operador que aún no había definido, y en la segunda ecuación asumió los vectores propios | X están completos.
¿Qué hay de malo en derivar la definición de impulso (representada en la base de la posición) como se hizo en los enlaces que proporcioné (usando el operador de traducción)? ¿El problema es solo que se limita a la presentación en la base del puesto?
Sí a tu segundo comentario. Ya hemos definido los operadores hermitianos X ^ y PAG ^ por su relación de conmutación [ X ^ , PAG ^ ] = i . Con esta definición y el postulado de QM que has planteado, todo sigue. Estoy disponible en el chat si lo desea.
Vale gracias, si nos quedamos sin comentarios lo moveremos al chat. ¿Qué quieres decir con que ya definimos los operadores? X ^ y PAG ^ por su relación de conmutación', ¿no obtienes la relación de conmutación de las definiciones de X ^ y PAG ^ ?
No. Empezamos con el sistema clásico con coordenadas X y momento canónico PAG que satisfacen un corchete de Poisson { X , PAG } = 1 y un espacio de fase METRO . La teoría cuántica se obtiene entonces definiendo un espacio de Hilbert H en el que cada estado corresponde a un punto en METRO . X y PAG son elevados a operadores X ^ y PAG ^ y los soportes de Poisson se elevan a los conmutadores. El resultado es que ahora tenemos un sistema cuántico descrito por un espacio de estados de Hilbert H y dos operadores X ^ y PAG ^ que satisfacen [ X ^ , PAG ^ ] = i .
Luego usamos el postulado de que los vectores propios de los operadores hermitianos forman una base en H y seguir adelante con la discusión que presenté en mi respuesta. Se puede elegir cualquier base y dependiendo de la elección de la base, los operadores X ^ y PAG ^ tienen diferentes representaciones. La más común es la base de coordenadas con la que está familiarizado. Otro común es la base propia de energía.
Una última pregunta. ¿Cuál es la motivación de la definición? X | pag = mi i pag X ? ¿De dónde viene?
También se deriva de la relación de conmutación [ X ^ , PAG ^ ] = i .
De acuerdo, ¿sabemos que los estados propios | X son continuos en lugar de discretos simplemente observando que la partícula se puede encontrar en cualquier lugar y no está limitada en su posición?
Sí. Eso es correcto.

Tenga en cuenta que X ^ = X no es estrictamente correcto. El lado izquierdo es el operador que actúa sobre los estados de Hilbert de los estados cuánticos. El lado derecho es un número real.

El RHS es la representación de posición del LHS. Como OP reconoció correctamente, esto es una tautología hasta cierto punto, ya que es la representación definitoria del operador a través de la ecuación de valor propio

X ^ | X = X | X .

Así que la definición a priori es esta con la noción de que el | X -los estados son funciones delta localizadas, es decir

X | X = d ( 3 ) ( X X ) .

Tenga en cuenta que la segunda ecuación anterior se deriva de la primera ecuación (hasta una constante) bajo el supuesto de que X ^ es hermítica y que su espectro es continuo.

Hasta ahora, esto fue solo algunas matemáticas, si desea leer sobre esto de una manera más precisa, le recomiendo Quantum Optics in Phase Space de W. Schleich .

Hasta ahora esto era sólo matemáticas. Lo importante que determina la física es la cuantización, que consiste en imponer la relación:

[ X ^ , PAG ^ ] = i

que es también cómo se obtiene la primera relación en la pregunta (es decir, el operador de cantidad de movimiento en la representación de posición).

Hace X | X = d ( X X ) sigue del siguiente razonamiento: X | X ^ | X = X | X ^ | X = X X | X = X X | X = X X | X . Esto vale para cualquier X , X por lo tanto debemos tener eso X | X = d ( X y ) ?
@JohnDoe sí, tienes razón. los dos últimos equiparados implican que es distinto de cero en todas partes excepto cuando x=x'. y luego, para obtener estados completos, necesita que sea una función delta (por eso dije "hasta una constante" en la pregunta; lo que acabo de mencionar corrige esa constante)
Entonces, solo para confirmar, ¿el hecho de que los operadores correspondientes a los observables sean hermticianos es un postulado en lugar de algo derivado?
@JohnDoe sí, de hecho. ese es en realidad uno de los axiomas de la mecánica cuántica. está vagamente motivado por el hecho de que los operadores no hermitianos pueden tener valores propios complejos, lo que no sería físico para los observables.

En QM tiene un postulado general, que requiere observables físicos (momento, momento angular, coordenadas) para ser representados por operadores hermitianos. En realidad, en QM tienes bases diferentes, así como una noción diferente de la evolución del tiempo. Esas son las bases de posición y coordenadas y las representaciones de Schrödinger y Heisenberg. Donde la diferencia entre los dos últimos es que en la imagen de Heisenberg los operadores dependen del tiempo. Su pregunta es para la base de coordenadas de representación de Schrödinger, así que concentrémonos en ella.

En la representación de coordenadas, la base está formada por un conjunto infinito de vectores | X > para lo cual se cumple lo siguiente

X ^ | X >= X | X >

Cada vector de estado | ψ > puede expresarse en esta base como
| ψ >= d X | X >< X | ψ > ,
donde usamos la expresión usual d X | X >< X | = I ^ . Entonces, el operador de posición fue fácil, ¿qué pasa con el operador de momento? Aquí necesitas más cálculo. De manera similar, para la representación del momento tenemos
PAG ^ | pag >= pag | pag >

y
| ψ >= d pag | pag >< pag | ψ > .
Para relacionar los dos anotamos:
ψ ( X ) =< X | ψ >= d pag < X | pag >< pag | ψ >= d pag < X | pag > ϕ ( pag ) ,
lo que significa que vas de una base a la otra a través de una transformación de Fourier. Donde la transformada de Fourier está dada por
ψ ( X ) = 1 ( 2 π ) 2 3 d pag mi i X pag ϕ ( pag )
En esta base, para el operador de cantidad de movimiento PAG ^ tenemos
< X | PAG ^ | ψ >=< X | PAG ^ d pag | pag >< pag | ψ >= 1 ( 2 π ) 2 3 d pag pag mi i X pag ϕ ( pag )
Desde
pag mi i X pag = i mi i X pag ,
tenemos
< X | PAG ^ | ψ >= i < X | ψ > .
O simplemente PAG ^ = i y para el otro operador X ^ | X >= X | X > por definición, porque estamos trabajando en la base de coordenadas.

Puedes comprobar fácilmente que ambos PAG ^ y X ^ son hermíticos y satisfacen la relación de Heisenberg.

PD También puede tener la imagen de Dirac (interacción) donde tanto los estados como los operadores dependen del tiempo.